ChaseDream
搜索
返回列表 发新帖
00:00:00

Editorial:In Ledland, unemployed adults receive government assistance. To reduce unemployment, the government proposes to supplement the income of those who accept jobs that pay less than government assistance, thus enabling employers to hire workers cheaply. However, the supplement will not raise any worker's income above what government assistance would provide if he or she were not gainfully employed. Therefore, unemployed people will have no financial incentive to accept jobs that would entitle them to the supplement.

Which of the following, if true about Ledland, most seriously weakens the argument of the editorial?

正确答案: C

更多相关帖子

524

帖子

15

好友

4712

积分

ChaseDream

注册时间
2003-03-17
精华
8
解析
查看: 1897|回复: 4
打印 上一主题 下一主题

GWD上的逻辑题,有点纠结

[复制链接]
跳转到指定楼层
楼主
发表于 2012-4-14 15:34:31 | 只看该作者 回帖奖励 |倒序浏览 |阅读模式
Editorial:In Ledland, unemployed adults receive government assistance.To reduce unemployment, the government proposes to supplement the income of those who accept jobs that pay less than government assistance, thus enabling employers to hire workers cheaply.However, the supplement will not raise any worker’s income above what government assistance would provide if he or she were not gainfully employed.Therefore, unemployed people will have no financial incentive to accept jobs that would entitle them to the supplement.

Which of the following, if true about Ledland, most seriously weakens the argument of the editorial?

A.The government collects no taxes on assistance it provides to unemployed individuals and their families.

B.Neighboring countries with laws that mandate the minimum wage an employer must pay an employee have higher unemployment rates than Ledland currently has.

C.People who are employed and look for a new job tend to get higher-paying jobs than job seekers who are unemployed.

D.The yearly amount unemployed people receive from government assistance is less than the yearly income that the government defines as the poverty level.

E.People sometimes accept jobs that pay relatively little simply because they enjoy the work.
这题答案是C选项,可是我觉得C有反对前提的嫌疑,因为原文中however后面的内容说不会高。有同学能给解释解释吗?感激不尽!
收藏收藏 收藏收藏
沙发
发表于 2012-4-14 18:01:35 | 只看该作者
You did not read carefully.  What is 不会高 in the passage and what is 会高 in (C)?
板凳
 楼主| 发表于 2012-4-14 18:06:51 | 只看该作者
谢谢!但是我还是有疑问,就是原文中说政府给worker的补助不会使这些worker的工资高于没有工作的人的补助,但是C选项相当于否定了该前提了
地板
发表于 2012-4-14 21:52:37 | 只看该作者
Read again.  Income does not equal salary . . .
5#
 楼主| 发表于 2012-4-15 08:41:07 | 只看该作者
理解了!感激
您需要登录后才可以回帖 登录 | 立即注册

Mark一下! 看一下! 顶楼主! 感谢分享! 快速回复:

手机版|ChaseDream|GMT+8, 2024-9-23 07:33
京公网安备11010202008513号 京ICP证101109号 京ICP备12012021号

ChaseDream 论坛

© 2003-2023 ChaseDream.com. All Rights Reserved.

返回顶部